Show that $mathscr{O}_{mathbb{Q}(sqrt{-7})}$ is a UFD












5












$begingroup$


It is known that the ring of integer is a Dedekind domain which means that it is a UFD iff it is a PID. Since $-7equiv1$ mod $4$, we have that $mathscr{O}_{mathbb{Q}(sqrt{-7})}=mathbb{Z}left[frac{1+sqrt{-7}}{2}right]$. Now I read something in the sense of: if $alpha:=frac{1+sqrt{-7}}{2}$ has an irreducible minimal polynomial mod $2$ and mod $3$, then we have a PID; I don't know anything about that. I think I have stated that wrong since the minimal polynomial is $f_{alpha}=x^2-x+2$ which is reducible mod $2$.



Dr. Math:
We pick an arbitrary complex number $x + iyinmathbb{Z}[alpha]$, and we must find a
suitable lattice point:



$$z = r + salpha = (r+s/2) + i(ssqrt{7})/2.$$



It is natural to try to have the real and imaginary parts of
$(x + yi - z)$ as small as possible.



Let's start with the imaginary part $ y - ssqrt{7}/2$. We take $s$ as
the closest integer to $2y/sqrt{7}$. This will give us the following:
begin{align*}
| 2y/sqrt{7} - s | &leqslant 1/2\
| y - ssqrt{7}/2 | &leqslant sqrt{7}/4.
end{align*}

Now, we turn to the real part $x - r - s/2$. If we select $r$ as the
integer closest to $(x - s/2)$, we will have:
begin{align*}
| x - r - s/2 | leqslant 1/2.
end{align*}

Putting both relations together, we get:
$$
N(x + yi - z) = (x - r - s/2)^2 + (y - ssqrt{7}/2)^2
leqslant 1/4 + 7/16
< 1$$



as desired. Hence, Euclidean domain, so PID, so UFD.



Is this proof correct and can it be applied in all cases of showing that $mathbb{Z}left[frac{1+sqrt{d}}{2}right]$, $squareneq dinmathbb{N}$ is a Euclidean domain?










share|cite|improve this question











$endgroup$








  • 1




    $begingroup$
    It's a Euclidean domain.
    $endgroup$
    – Lord Shark the Unknown
    Dec 16 '18 at 20:22






  • 1




    $begingroup$
    @LordSharktheUnknown Yes, but how can I show that? Is there a method for showing such?
    $endgroup$
    – Algebear
    Dec 16 '18 at 20:26










  • $begingroup$
    I didn't find the exact place yet but the conclusion that all $mathbb Z[ 1+sqrt d / 2]$ for $d equiv 1 pmod 4$ are euclidean is false. Something like this should work for $7$ i think, but there should be some part that breaks down once $d$ is large.
    $endgroup$
    – Alex J Best
    Dec 16 '18 at 21:10












  • $begingroup$
    @AlexJBest: You're right and an example for what you say is $ d = -19 $.
    $endgroup$
    – hellHound
    Dec 17 '18 at 17:47










  • $begingroup$
    The Minkowski bound for $Bbb Q(sqrt{-7})$ is ~1.68..., if you have this machinery available to you.
    $endgroup$
    – ÍgjøgnumMeg
    Dec 22 '18 at 11:30
















5












$begingroup$


It is known that the ring of integer is a Dedekind domain which means that it is a UFD iff it is a PID. Since $-7equiv1$ mod $4$, we have that $mathscr{O}_{mathbb{Q}(sqrt{-7})}=mathbb{Z}left[frac{1+sqrt{-7}}{2}right]$. Now I read something in the sense of: if $alpha:=frac{1+sqrt{-7}}{2}$ has an irreducible minimal polynomial mod $2$ and mod $3$, then we have a PID; I don't know anything about that. I think I have stated that wrong since the minimal polynomial is $f_{alpha}=x^2-x+2$ which is reducible mod $2$.



Dr. Math:
We pick an arbitrary complex number $x + iyinmathbb{Z}[alpha]$, and we must find a
suitable lattice point:



$$z = r + salpha = (r+s/2) + i(ssqrt{7})/2.$$



It is natural to try to have the real and imaginary parts of
$(x + yi - z)$ as small as possible.



Let's start with the imaginary part $ y - ssqrt{7}/2$. We take $s$ as
the closest integer to $2y/sqrt{7}$. This will give us the following:
begin{align*}
| 2y/sqrt{7} - s | &leqslant 1/2\
| y - ssqrt{7}/2 | &leqslant sqrt{7}/4.
end{align*}

Now, we turn to the real part $x - r - s/2$. If we select $r$ as the
integer closest to $(x - s/2)$, we will have:
begin{align*}
| x - r - s/2 | leqslant 1/2.
end{align*}

Putting both relations together, we get:
$$
N(x + yi - z) = (x - r - s/2)^2 + (y - ssqrt{7}/2)^2
leqslant 1/4 + 7/16
< 1$$



as desired. Hence, Euclidean domain, so PID, so UFD.



Is this proof correct and can it be applied in all cases of showing that $mathbb{Z}left[frac{1+sqrt{d}}{2}right]$, $squareneq dinmathbb{N}$ is a Euclidean domain?










share|cite|improve this question











$endgroup$








  • 1




    $begingroup$
    It's a Euclidean domain.
    $endgroup$
    – Lord Shark the Unknown
    Dec 16 '18 at 20:22






  • 1




    $begingroup$
    @LordSharktheUnknown Yes, but how can I show that? Is there a method for showing such?
    $endgroup$
    – Algebear
    Dec 16 '18 at 20:26










  • $begingroup$
    I didn't find the exact place yet but the conclusion that all $mathbb Z[ 1+sqrt d / 2]$ for $d equiv 1 pmod 4$ are euclidean is false. Something like this should work for $7$ i think, but there should be some part that breaks down once $d$ is large.
    $endgroup$
    – Alex J Best
    Dec 16 '18 at 21:10












  • $begingroup$
    @AlexJBest: You're right and an example for what you say is $ d = -19 $.
    $endgroup$
    – hellHound
    Dec 17 '18 at 17:47










  • $begingroup$
    The Minkowski bound for $Bbb Q(sqrt{-7})$ is ~1.68..., if you have this machinery available to you.
    $endgroup$
    – ÍgjøgnumMeg
    Dec 22 '18 at 11:30














5












5








5


2



$begingroup$


It is known that the ring of integer is a Dedekind domain which means that it is a UFD iff it is a PID. Since $-7equiv1$ mod $4$, we have that $mathscr{O}_{mathbb{Q}(sqrt{-7})}=mathbb{Z}left[frac{1+sqrt{-7}}{2}right]$. Now I read something in the sense of: if $alpha:=frac{1+sqrt{-7}}{2}$ has an irreducible minimal polynomial mod $2$ and mod $3$, then we have a PID; I don't know anything about that. I think I have stated that wrong since the minimal polynomial is $f_{alpha}=x^2-x+2$ which is reducible mod $2$.



Dr. Math:
We pick an arbitrary complex number $x + iyinmathbb{Z}[alpha]$, and we must find a
suitable lattice point:



$$z = r + salpha = (r+s/2) + i(ssqrt{7})/2.$$



It is natural to try to have the real and imaginary parts of
$(x + yi - z)$ as small as possible.



Let's start with the imaginary part $ y - ssqrt{7}/2$. We take $s$ as
the closest integer to $2y/sqrt{7}$. This will give us the following:
begin{align*}
| 2y/sqrt{7} - s | &leqslant 1/2\
| y - ssqrt{7}/2 | &leqslant sqrt{7}/4.
end{align*}

Now, we turn to the real part $x - r - s/2$. If we select $r$ as the
integer closest to $(x - s/2)$, we will have:
begin{align*}
| x - r - s/2 | leqslant 1/2.
end{align*}

Putting both relations together, we get:
$$
N(x + yi - z) = (x - r - s/2)^2 + (y - ssqrt{7}/2)^2
leqslant 1/4 + 7/16
< 1$$



as desired. Hence, Euclidean domain, so PID, so UFD.



Is this proof correct and can it be applied in all cases of showing that $mathbb{Z}left[frac{1+sqrt{d}}{2}right]$, $squareneq dinmathbb{N}$ is a Euclidean domain?










share|cite|improve this question











$endgroup$




It is known that the ring of integer is a Dedekind domain which means that it is a UFD iff it is a PID. Since $-7equiv1$ mod $4$, we have that $mathscr{O}_{mathbb{Q}(sqrt{-7})}=mathbb{Z}left[frac{1+sqrt{-7}}{2}right]$. Now I read something in the sense of: if $alpha:=frac{1+sqrt{-7}}{2}$ has an irreducible minimal polynomial mod $2$ and mod $3$, then we have a PID; I don't know anything about that. I think I have stated that wrong since the minimal polynomial is $f_{alpha}=x^2-x+2$ which is reducible mod $2$.



Dr. Math:
We pick an arbitrary complex number $x + iyinmathbb{Z}[alpha]$, and we must find a
suitable lattice point:



$$z = r + salpha = (r+s/2) + i(ssqrt{7})/2.$$



It is natural to try to have the real and imaginary parts of
$(x + yi - z)$ as small as possible.



Let's start with the imaginary part $ y - ssqrt{7}/2$. We take $s$ as
the closest integer to $2y/sqrt{7}$. This will give us the following:
begin{align*}
| 2y/sqrt{7} - s | &leqslant 1/2\
| y - ssqrt{7}/2 | &leqslant sqrt{7}/4.
end{align*}

Now, we turn to the real part $x - r - s/2$. If we select $r$ as the
integer closest to $(x - s/2)$, we will have:
begin{align*}
| x - r - s/2 | leqslant 1/2.
end{align*}

Putting both relations together, we get:
$$
N(x + yi - z) = (x - r - s/2)^2 + (y - ssqrt{7}/2)^2
leqslant 1/4 + 7/16
< 1$$



as desired. Hence, Euclidean domain, so PID, so UFD.



Is this proof correct and can it be applied in all cases of showing that $mathbb{Z}left[frac{1+sqrt{d}}{2}right]$, $squareneq dinmathbb{N}$ is a Euclidean domain?







number-theory ring-theory algebraic-number-theory unique-factorization-domains euclidean-domain






share|cite|improve this question















share|cite|improve this question













share|cite|improve this question




share|cite|improve this question








edited Dec 17 '18 at 1:42









Batominovski

33.1k33293




33.1k33293










asked Dec 16 '18 at 20:21









AlgebearAlgebear

704419




704419








  • 1




    $begingroup$
    It's a Euclidean domain.
    $endgroup$
    – Lord Shark the Unknown
    Dec 16 '18 at 20:22






  • 1




    $begingroup$
    @LordSharktheUnknown Yes, but how can I show that? Is there a method for showing such?
    $endgroup$
    – Algebear
    Dec 16 '18 at 20:26










  • $begingroup$
    I didn't find the exact place yet but the conclusion that all $mathbb Z[ 1+sqrt d / 2]$ for $d equiv 1 pmod 4$ are euclidean is false. Something like this should work for $7$ i think, but there should be some part that breaks down once $d$ is large.
    $endgroup$
    – Alex J Best
    Dec 16 '18 at 21:10












  • $begingroup$
    @AlexJBest: You're right and an example for what you say is $ d = -19 $.
    $endgroup$
    – hellHound
    Dec 17 '18 at 17:47










  • $begingroup$
    The Minkowski bound for $Bbb Q(sqrt{-7})$ is ~1.68..., if you have this machinery available to you.
    $endgroup$
    – ÍgjøgnumMeg
    Dec 22 '18 at 11:30














  • 1




    $begingroup$
    It's a Euclidean domain.
    $endgroup$
    – Lord Shark the Unknown
    Dec 16 '18 at 20:22






  • 1




    $begingroup$
    @LordSharktheUnknown Yes, but how can I show that? Is there a method for showing such?
    $endgroup$
    – Algebear
    Dec 16 '18 at 20:26










  • $begingroup$
    I didn't find the exact place yet but the conclusion that all $mathbb Z[ 1+sqrt d / 2]$ for $d equiv 1 pmod 4$ are euclidean is false. Something like this should work for $7$ i think, but there should be some part that breaks down once $d$ is large.
    $endgroup$
    – Alex J Best
    Dec 16 '18 at 21:10












  • $begingroup$
    @AlexJBest: You're right and an example for what you say is $ d = -19 $.
    $endgroup$
    – hellHound
    Dec 17 '18 at 17:47










  • $begingroup$
    The Minkowski bound for $Bbb Q(sqrt{-7})$ is ~1.68..., if you have this machinery available to you.
    $endgroup$
    – ÍgjøgnumMeg
    Dec 22 '18 at 11:30








1




1




$begingroup$
It's a Euclidean domain.
$endgroup$
– Lord Shark the Unknown
Dec 16 '18 at 20:22




$begingroup$
It's a Euclidean domain.
$endgroup$
– Lord Shark the Unknown
Dec 16 '18 at 20:22




1




1




$begingroup$
@LordSharktheUnknown Yes, but how can I show that? Is there a method for showing such?
$endgroup$
– Algebear
Dec 16 '18 at 20:26




$begingroup$
@LordSharktheUnknown Yes, but how can I show that? Is there a method for showing such?
$endgroup$
– Algebear
Dec 16 '18 at 20:26












$begingroup$
I didn't find the exact place yet but the conclusion that all $mathbb Z[ 1+sqrt d / 2]$ for $d equiv 1 pmod 4$ are euclidean is false. Something like this should work for $7$ i think, but there should be some part that breaks down once $d$ is large.
$endgroup$
– Alex J Best
Dec 16 '18 at 21:10






$begingroup$
I didn't find the exact place yet but the conclusion that all $mathbb Z[ 1+sqrt d / 2]$ for $d equiv 1 pmod 4$ are euclidean is false. Something like this should work for $7$ i think, but there should be some part that breaks down once $d$ is large.
$endgroup$
– Alex J Best
Dec 16 '18 at 21:10














$begingroup$
@AlexJBest: You're right and an example for what you say is $ d = -19 $.
$endgroup$
– hellHound
Dec 17 '18 at 17:47




$begingroup$
@AlexJBest: You're right and an example for what you say is $ d = -19 $.
$endgroup$
– hellHound
Dec 17 '18 at 17:47












$begingroup$
The Minkowski bound for $Bbb Q(sqrt{-7})$ is ~1.68..., if you have this machinery available to you.
$endgroup$
– ÍgjøgnumMeg
Dec 22 '18 at 11:30




$begingroup$
The Minkowski bound for $Bbb Q(sqrt{-7})$ is ~1.68..., if you have this machinery available to you.
$endgroup$
– ÍgjøgnumMeg
Dec 22 '18 at 11:30










0






active

oldest

votes











Your Answer





StackExchange.ifUsing("editor", function () {
return StackExchange.using("mathjaxEditing", function () {
StackExchange.MarkdownEditor.creationCallbacks.add(function (editor, postfix) {
StackExchange.mathjaxEditing.prepareWmdForMathJax(editor, postfix, [["$", "$"], ["\\(","\\)"]]);
});
});
}, "mathjax-editing");

StackExchange.ready(function() {
var channelOptions = {
tags: "".split(" "),
id: "69"
};
initTagRenderer("".split(" "), "".split(" "), channelOptions);

StackExchange.using("externalEditor", function() {
// Have to fire editor after snippets, if snippets enabled
if (StackExchange.settings.snippets.snippetsEnabled) {
StackExchange.using("snippets", function() {
createEditor();
});
}
else {
createEditor();
}
});

function createEditor() {
StackExchange.prepareEditor({
heartbeatType: 'answer',
autoActivateHeartbeat: false,
convertImagesToLinks: true,
noModals: true,
showLowRepImageUploadWarning: true,
reputationToPostImages: 10,
bindNavPrevention: true,
postfix: "",
imageUploader: {
brandingHtml: "Powered by u003ca class="icon-imgur-white" href="https://imgur.com/"u003eu003c/au003e",
contentPolicyHtml: "User contributions licensed under u003ca href="https://creativecommons.org/licenses/by-sa/3.0/"u003ecc by-sa 3.0 with attribution requiredu003c/au003e u003ca href="https://stackoverflow.com/legal/content-policy"u003e(content policy)u003c/au003e",
allowUrls: true
},
noCode: true, onDemand: true,
discardSelector: ".discard-answer"
,immediatelyShowMarkdownHelp:true
});


}
});














draft saved

draft discarded


















StackExchange.ready(
function () {
StackExchange.openid.initPostLogin('.new-post-login', 'https%3a%2f%2fmath.stackexchange.com%2fquestions%2f3043108%2fshow-that-mathscro-mathbbq-sqrt-7-is-a-ufd%23new-answer', 'question_page');
}
);

Post as a guest















Required, but never shown

























0






active

oldest

votes








0






active

oldest

votes









active

oldest

votes






active

oldest

votes
















draft saved

draft discarded




















































Thanks for contributing an answer to Mathematics Stack Exchange!


  • Please be sure to answer the question. Provide details and share your research!

But avoid



  • Asking for help, clarification, or responding to other answers.

  • Making statements based on opinion; back them up with references or personal experience.


Use MathJax to format equations. MathJax reference.


To learn more, see our tips on writing great answers.




draft saved


draft discarded














StackExchange.ready(
function () {
StackExchange.openid.initPostLogin('.new-post-login', 'https%3a%2f%2fmath.stackexchange.com%2fquestions%2f3043108%2fshow-that-mathscro-mathbbq-sqrt-7-is-a-ufd%23new-answer', 'question_page');
}
);

Post as a guest















Required, but never shown





















































Required, but never shown














Required, but never shown












Required, but never shown







Required, but never shown

































Required, but never shown














Required, but never shown












Required, but never shown







Required, but never shown







Popular posts from this blog

Le Mesnil-Réaume

Ida-Boy-Ed-Garten

web3.py web3.isConnected() returns false always